Convert a cylindrical coordinate vector to cartesian coordinates

Click For Summary
The discussion focuses on converting a cylindrical coordinate vector to Cartesian coordinates using the provided equations. The user successfully calculates the components A_x, A_y, and A_z based on the cylindrical coordinates and combines them into a Cartesian vector. The final expression includes terms involving x, y, z, and the square root of x^2 + y^2. The user seeks confirmation on the correctness of their conversion and whether further simplification is possible. The conversation emphasizes the importance of accuracy in vector transformations in physics.
VinnyCee
Messages
486
Reaction score
0

Homework Statement



Convert the following cylindrical coordinate vector to a Cartesian vector:

\overrightarrow{A}\,=\,\rho\,z\,sin\,\phi\,\hat{\rho}\,+\,3\,\rho\,cos\,\phi\,\hat{\phi}\,+\,\rho\,cos\,\phi\,sin\,\phi\,\hat{z}



Homework Equations



A_x\,=\,\hat{x}\,\cdot\,\overrightarrow{A}\,=\,\left(\hat{x}\,\cdot\,\hat{\rho}\right)\,A_{\rho}\,+\,\left(\hat{x}\,\cdot\,\hat{\phi}\right)\,A_{\phi}\,+\,\left(\hat{x}\,\cdot\,\hat{z}\right)\,A_{z}\,=\,A_{\rho}\,cos\,\phi\,-\,A_{\phi}\,sin\,\phi

Following the steps in the above equation...

A_y\,=\,A_{\rho}\,sin\,\phi\,+\,A_{\phi}\,cos\,\phi

A_z\,=\,A_z

Also, use these definitions after one completes initial conversion using the equations above...

cos\,\phi\,=\,\frac{x}{\rho}

sin\,\phi\,=\,\frac{y}{\rho}

\rho^2\,=\,x^2\,+\,y^2



The Attempt at a Solution



Using the above equations for A_x, A_y and A_z, I get:

A_x\,=\,\rho\,z\,cos\,\phi\,sin\,\phi\,-\,3\,\rho\,cos\,\phi\,sin\,\phi

A_y\,=\,\rho\,z\,sin^2\,\phi\,+\,3\,\rho\,cos^2\,\phi

A_z\,=\,\rho\,cos\,\phi\,sin\,\phi

Now combine into a vector...

\overrightarrow{A}\,=\,\hat{x}\,\left(\rho\,z\,cos\,\phi\,sin\,\phi\,-\,3\,\rho\,cos\,\phi\,sin\,\phi\right)\,+\,\hat{y}\,\left(\rho\,z\,sin^2\,\phi\,+\,3\,\rho\,cos^2\,\phi\right)\,+\,\hat{z}\,\left(\rho\,cos\,\phi\,sin\,\phi\right)

Using the bottom three definitions in the Relevant Equations section above...

\overrightarrow{A}\,=\,\hat{x}\,\left(x\,z\,\frac{y}{\rho}\right)\,+\,\hat{y}\,\left(y\,z\,\frac{y}{\rho}\,+\,3\,x\,\frac{x}{\rho}\right)\,+\,\hat{z}\,\left(x\,\frac{y}{\rho}\right)

\overrightarrow{A}\,=\,\hat{x}\,\left(\frac{x\,y\,z}{\sqrt{x^2\,+\,y^2}}\,-\,\frac{3\,x\,y}{\sqrt{x^2\,+\,y^2}}\right)\,+\,\hat{y}\,\left(\frac{y^2\,z}{\sqrt{x^2\,+\,y^2}}\,+\,\frac{3\,x^2}{\sqrt{x^2\,+\,y^2}}\right)\,+\,\hat{z}\,\left(\frac{x\,y}{\sqrt{x^2\,+\,y^2}}\right)

\overrightarrow{A}\,=\,\hat{x}\,\left(\frac{x\,y\,z\,-\,3\,x\,y}{\sqrt{x^2\,+\,y^2}}\right)\,+\,\hat{y}\,\left(\frac{y^2\,z\,+\,3\,x^2}{\sqrt{x^2\,+\,y^2}}\right)\,+\,\hat{z}\,\left(\frac{x\,y}{\sqrt{x^2\,+\,y^2}}\right)

Does that seem right, or is there more simplification that can be done?
 
Physics news on Phys.org
It looks right, but I just want to make sure it is before going on to the rest of the problems like this one!
 
Question: A clock's minute hand has length 4 and its hour hand has length 3. What is the distance between the tips at the moment when it is increasing most rapidly?(Putnam Exam Question) Answer: Making assumption that both the hands moves at constant angular velocities, the answer is ## \sqrt{7} .## But don't you think this assumption is somewhat doubtful and wrong?

Similar threads

  • · Replies 7 ·
Replies
7
Views
2K
  • · Replies 2 ·
Replies
2
Views
2K
  • · Replies 5 ·
Replies
5
Views
2K
Replies
2
Views
1K
Replies
3
Views
2K
  • · Replies 34 ·
2
Replies
34
Views
3K
  • · Replies 4 ·
Replies
4
Views
2K
  • · Replies 1 ·
Replies
1
Views
2K
  • · Replies 2 ·
Replies
2
Views
2K